Go Back   StudyChaCha 2024 2025 > StudyChaCha Discussion Forum > General Topics

  #1  
Old April 25th, 2013, 12:57 PM
Super Moderator
 
Join Date: Dec 2011
Default LSAT Test Questions and Answers

I want to give the exam of LSAT Test so I want to get some questions and answers so can you provide me that

Here is the list of few LSAT Logical Reasoning Practice Questions which you are looking for.

Question 1
If all beaches were publicly owned, we would have to rely on government funds to maintain them. It is true that more people would have access to the ocean and beaches, but at what cost? If the beaches are not cared for adequately, soon there will be nothing left worth having access to. We should consider carefully before nationalizing more coastal property.
Which of the following, if true, would most weaken the argument above?

1. The public does not want additional access to beaches.
2. The government is currently responsible for the maintenance of all public and private beaches.
3. The public already has some access to many beaches.
4. Other property has been nationalized in the past with no complaints from the original owners of the property.
5. Some privately owned beaches are not well maintained.

Question 2
A recent university study indicates that students who receive full scholarships tend to maintain higher grade point averages than do students who must take out loans or work to finance school. The survey concluded that scholarships enable students to achieve high grade point averages by alleviating the stress related to financial concerns and freeing up students' time to study more.
The conclusion drawn in the argument above follows logically if which one of the following is assumed?

1. Students who take out loans maintain higher grade point averages than those who work to finance school.

2. Finance-related stress affects student performance in a manner similar to that of restricted study time.

3. Students who must work to pay for their studies cannot maintain high grade point averages.

4. High grade point averages were not the primary criterion upon which the scholarship awards were based.

5. Controlling stress level is less important to student performance than is intensive studying.

Question 3
Plant Y thrives in environments of great sunlight and very little moisture. Desert X is an environment with constant, powerful sunlight, and next to no moisture. Although Plant Y thrives in the areas surrounding Desert X, it does not exist naturally in the desert, nor does it survive long when introduced there.
Which of the following would be most useful in explaining the apparent discrepancy above?

1. Desert X's climate is far too harsh for the animals that normally feed on Plant Y.

2. For one week in the fall, Desert X gets consistent rainfall.

3. The environment around Desert X is ideally suited to the needs of Plant Y.

4. Due to the lack of sufficient moisture, Desert X can support almost no plant life.

5. Plant Y cannot survive in temperatures as high as those normally found in Desert X.

Question 4
Opening a plant in war-torn Country X is not inadvisable, despite what critics of the plan may say. Ten years ago we opened our plant in Country Y in the middle of a revolution; that plant has been generating successful profits ever since.
Which of the following is the author of the argument above most reasonably intending the reader to conclude?

1. Wars are profitable for the author's particular business.
2. Country X is a more stable nation than Country Y.
3. Critics of the proposed plant in Country Y are likely to be biased.
4. The proposed plant in Country X will generate profits, despite war.
5. The proposed plant in Country X will be more successful than in Country Y.

Question 5
The cost of living in a given area is directly proportional to the wages paid by the dominant industry in that area. In Moomba, the dominant industry is farming. However, in Sepra, the dominant industry is auto manufacturing. Thus, it is fair to conclude that the cost of living must be higher in Sepra than in Moomba.
The argument above assumes that

1. the quality of life in Sepra is higher than the quality of life in Moomba.

2. the number of people manufacturing autos in Moomba is higher than the number of people manufacturing autos in Sepra.

3. the cost of living in Sepra is higher than the cost of living in any other area.

4. wages paid by the auto manufacturing industry are higher than those of the farming industry.

5. automobile workers in Sepra are likely to leave Sepra and move to areas with a lower cost of living.

Last edited by Aakashd; February 26th, 2020 at 09:14 AM.
Reply With Quote
Other Discussions related to this topic
Thread
Aptitude Test Questions And Answers For Management Trainee
PSC Test Questions Answers
NIIT aptitude test questions and answers
HDFC Bank Written Test Questions And Answers
How Many Questions on the LSAT
Inference Questions Lsat
LSAT CR Questions
SAT Test Questions and Answers
LSAT Practice Test Questions
LSAT Test Questions Practice
LSAT Test Questions Logic
Kaplan LSAT Diagnostic Test Answers
LSAT CR questions
LSAT Test Questions Free
MBA Sample Test Comprehensive Questions With Answers
Test Questions And Answers For Citizenship Test
SSC Test Questions And Answers
LSAT online practice test questions
LSAT Questions and Answers
LSAT Logical Reasoning Questions and Answers






  #2  
Old April 26th, 2013, 11:37 AM
Super Moderator
 
Join Date: Nov 2011
Default Re: LSAT Test Questions and Answers

As you want to get the some questions and answers of LSAT Test so here is the information of the same:

Some content of the file has been given here:

Directions: Each group of questions in this section is based on a set of conditions. In answering some of the questions, it may be useful to draw a rough diagram. Choose the response that most accurately and completely answers each question and blacken the corresponding space on your answer sheet.

Questions 1–5
A company employee generates a series of five-digit product codes in accordance with the following rules:
The codes use the digits 0, 1, 2, 3, and 4, and no others. Each digit occurs exactly once in any code.
The second digit has a value exactly twice that of the first digit.
The value of the third digit is less than the value of the fifth digit.

1. If the last digit of an acceptable product code is 1, it must be true that the
(A) first digit is 2
(B) second digit is 0
(C) third digit is 3
(D) fourth digit is 4
(E) fourth digit is 0

2. Which one of the following must be true about any acceptable product code?
(A) The digit 1 appears in some position before the digit 2.
(B) The digit 1 appears in some position before the digit 3.
(C) The digit 2 appears in some position before the digit 3.
(D) The digit 3 appears in some position before the digit 0.
(E) The digit 4 appears in some position before the digit 3.

3. If the third digit of an acceptable product code is not 0, which one of the following must be true?
(A) The second digit of the product code is 2.
(B) The third digit of the product code is 3.
(C) The fourth digit of the product code is 0.
(D) The fifth digit of the product code is 3.
(E) The fifth digit of the product code is 1.

For more detailed information I am uploading a PDF file which is free to download:
Attached Files Available for Download
File Type: doc List of MBA colleges in Greater Noida.doc (62.0 KB, 52 views)
__________________
Answered By StudyChaCha Member
Reply With Quote
  #3  
Old December 24th, 2014, 09:22 AM
Unregistered
Guest
 
Default Re: LSAT Test Questions and Answers

Will you please provide the list of LSAT Logical Reasoning Practice Questions?
Reply With Quote
Reply




All times are GMT +6. The time now is 09:20 AM.


Powered by vBulletin® Version 3.8.11
Copyright ©2000 - 2024, vBulletin Solutions Inc.
Search Engine Friendly URLs by vBSEO 3.6.0 PL2

1 2 3 4 5 6 7 8